Schwebungsfrequenz für 3 Wellen

Stellen Sie sich 3 Wellen mit einer Frequenz von 101, 103, 106 Hz und gleicher Intensität vor. Wie hoch sollten die Schwebungsfrequenzen sein?

Jetzt kann ich es für 2 Wellen berechnen, und ich weiß, wie man die kombinierte Gleichung der beiden schreibt. Aber das Hinzufügen einer dritten verursacht viele Probleme ... die Gleichung wird umständlich. Können Sie mir hier bitte helfen? Ich habe online herumgegraben, und einige Antworten haben die dritte Welle vernachlässigt, da sie zu nahe an einer der Wellen war. Wenn ich das vermeiden möchte, gibt es eine andere Lösung?

Eine schnelle Überprüfung von WolframAlpha ergab keine einfache Reduktion, die eine Überschreitung für cos (x) + cos (y) + cos (z) zeigte. Ich glaube, dass die Lösung umständlich sein wird, wie Sie sagen. Das mathematische Problem scheint zu sein, dass die Amplitude nach dem Kombinieren der ersten beiden Terme anders ist als die des dritten. Hörbar nähern Sie sich einem Chorus-Effekt. Ich habe ein kurzes Python-Programm gepostet, das eine Wave-Datei für 3 Sinuswellen generiert, wenn Sie hören möchten: physical.stackexchange.com/q/159182
Sie erhalten eine Schwebungsfrequenz für jedes Eingangspaar. Schreiben Sie einfach die vollständige Gleichung auf und trennen Sie die Terme.

Antworten (2)

Ein einfaches Python-Skript gibt Aufschluss - hier ist, was ich für die Summe der drei Häufigkeiten berechne:

Geben Sie hier die Bildbeschreibung ein

Es gibt offensichtlich eine starke Schwebung bei 1 Hz - dann sind alle drei Frequenzen (wieder) in Phase. Dazwischen gibt es kleinere Spitzen - von denen ich 5 Hz als die sichtbarste Komponente betrachten würde.

Dies entspricht der Intuition - Sie würden erwarten, dass die Differenzfrequenzen von 2 Hz, 3 Hz und vielleicht 5 Hz auftauchen, aber sie werden durcheinander gebracht. Und tatsächlich, 5 Hz sind da; die zwei größeren Spitzen um 0,5 s zeigen das 2-Hz-Signal an; während das 3-Hz-Signal nicht separat gesehen werden kann.

Hier ist der Code, den ich verwendet habe, um dieses Diagramm zu zeichnen:

# beat frequencies
import numpy as np
import matplotlib.pyplot as plt
from math import pi

f = [101,103,106];
Ns = 8*1024
t = np.linspace(0,2,Ns);
a = np.zeros((Ns),'double')
for fi in f:
    a = a + np.cos(2*pi*fi*t)

plt.figure(figsize=(10,5))
plt.plot(t, a)
plt.title('three frequencies beating')
plt.xlabel('time (s)')
plt.ylabel('amplitude')
plt.show()

Die Wellen mit separat erzeugen Beat mit Frequenzen von 2 , 3 Und 5 H z .

Diese Beats haben Zeitspannen 1 / 2 , 1 / 3 , Und 1 / 5 S , und nach diesen Zeitintervallen sind Schläge zu hören. Ihr LCM ist 1 S . So danach 1 S , hat die gesamte Welle die gleiche Phase und Amplitude, und es gibt einen starken Schlag. Die Frequenz dieses starken Schlags ist 1 H z .

Siehe die andere Antwort für eine numerische Überprüfung.

Eine eng verwandte Frage von JEE Mains. Die Zeiträume ergeben sich 1 , 1 , 1 / 2 dessen LCM ist 1 , aber es scheint nicht ganz die Antwort zu sein, die sie erwarten